LSAT and Law School Admissions Forum

Get expert LSAT preparation and law school admissions advice from PowerScore Test Preparation.

User avatar
 Dave Killoran
PowerScore Staff
  • PowerScore Staff
  • Posts: 5853
  • Joined: Mar 25, 2011
|
#55800
Complete Question Explanation
(The complete setup for this game can be found here: lsat/viewtopic.php?t=23743)

The correct answer choice is (B)

When Patterson meets with T at 4:00, then from the super-sequence we know that S, W, and Y must meet with Patterson prior to 4:00, with S at 1:00, and with W and Y rotating between 2:00 and 3:00:


pt45_d04_g1_q5a.png
Placing S, T, W, and Y forces U and R into the final two meetings, with U at 5:00 and R at 6:00:

Since this is a Must be true question, you should look for answer choices containing fixed variables (such as S, T, U, and R) and avoid answer choices that contain variables with uncertainty (W and Y).


pt45_d04_g1_q5b.png

Answer choice (A): This answer choice is incorrect because R must meet with Patterson at 6:00.

Answer choice (B): This is the correct answer choice.

Answer choice (C): This answer choice is incorrect because Patterson can meet with Y at 2:00 or 3:00.

Answer choice (D): This answer choice is incorrect because Patterson can meet with Y at 2:00 or 3:00.

Answer choice (E): This answer choice is incorrect because Patterson can workout at 2:00 or 3:00.
You do not have the required permissions to view the files attached to this post.

Get the most out of your LSAT Prep Plus subscription.

Analyze and track your performance with our Testing and Analytics Package.